General Awareness - SPLessons

UPSC Prelims Practice Sets

Home > > Tutorial
SPLessons 5 Steps, 3 Clicks
5 Steps - 3 Clicks

UPSC Prelims Practice Sets

shape Introduction

In General Current Affair means General Issue. These Current Affairs related to several fields like International, National, Sports, Politics, Economy, Science and Technology, and Environment. Current Affairs Section is very important in several exams like UPSC, SSC, Banking, RRB, SBI, IAS, and Insurance etc. Through daily newspapers and daily news, there is a huge chance of scoring maximum marks in several exams.
The main aim of the article UPSC Prelims Practice Sets is to clear the civil exams. The Civil's Exam is the toughest exam in competitive Field. It provides important and useful questions related to Current Affairs.

shape Quiz

1. 1. Indian Space Research Organisation (ISRO) and Russia’s Roscosmos State Corporation for Space Activities (ROSCOSMOS) have signed a MoU to work together for Gaganyaan. Consider the following about it. 1. Gangayaan is India’s first manned space mission. 2. Under the agreement, Indian astronauts will receive training at the International Space Station (ISS). 3. India would be the third nation to do so, if it is successful, after the United States, and Russia to have a manned space mission. Select the correct answer using the codes below.
    A. 1 and 2 only B. 2 and 3 only C. 1 only D. 1, 2 and 3 only

Answer - Option A
Explanation:
Justification: It is India’s first manned space mission. Under it, India is planning to send three humans (Gaganyatris) into space i.e. in low earth orbit (LEO) by 2022 i.e. by 75th Independence Day for period of five to seven days.
India plans to build a crew vehicle that can accommodate 2 or 3 astronauts and human rate its GLSV Mk-III launcher.
As per the MoU, ROSCOSMOS has offered ride to Indian astronaut short visit to International Space Station (ISS) on board Soyuz spacecraft for short training mission in 2022.
A manned space mission is very different from all other missions that ISRO has so far completed. In terms of complexity and ambition, even the missions to the Moon (Chandrayaan) and Mars (Mangalyaan) are nowhere in comparison.
For a manned mission, the key distinguishing capabilities that ISRO has had to develop include the ability to bring the spacecraft back to Earth after flight, and to build a spacecraft in which astronauts can live in Earth-like conditions in space.
If India does launch the Gaganyaan mission, it will be the the fourth nation to do so after the United States, Russia and China.
In what appears to be a preparation for the Gaganyaan mission, ISRO recently conducted its first ‘pad abort’ test that was successful.
The ‘pad abort’ test or Crew Escape System is an emergency escape measure that helps pull the crew away from the launch vehicle when a mission has to be aborted. The test was conducted at the Satish Dhawan Space Centre, Sriharikota.
The Pad Abort Test demonstrated the safe recovery of the crew module in case of any exigency at the launch pad.
2. The Ganges River dolphin, or susu, inhabits which of the following river systems apart from Ganges? 1. Brahmaputra 2. Meghna 3. Krishna Select the correct answer using the codes below
    A. 1 only B. 2 only C. 1 and 2 only D. 2 and 3 only

Answer - Option C
Explanation:
Justification: They inhabit the Ganges-Brahmaputra-Meghna and Karnaphuli-Sangu river systems of Nepal, India, and Bangladesh.
It is classified as endangered by the IUCN.
This dolphin is among the four “obligate” freshwater dolphins – the other three are the baiji now likely extinct from the Yangtze river in China, the bhulan of the Indus in Pakistan and the boto of the Amazon River in Latin America.
Although there are several species of marine dolphins whose ranges include some freshwater habitats, these four species live only in rivers and lakes.
Being a mammal, the Ganges River dolphin cannot breathe in the water and must surface every 30-120 seconds.
Because of the sound it produces when breathing, the animal is popularly referred to as the ‘Susu’.
Learning: The survival of the Ganges River dolphin is threatened by unintentional killing through entanglement in fishing gear; directed harvest for dolphin oil, which is used as a fish attractant and for medicinal purposes; water development projects (e.g. water extraction and the construction of barrages, high dams, and embankments); industrial waste and pesticides; municipal sewage discharge and noise from vessel traffic; and over exploitation of prey, mainly due to the widespread use of non selective fishing gear.
India’s and Asia’s first Dolphin Research Centre will be set up on the banks of the Ganga river in Patna University campus in Patna, Bihar. It will be named- National Dolphin Research Centre (NDRC).
The announcement for the centre was made on the occasion of Dolphin day (October 5), observed in Bihar for protection and conservation of Gangetic river dolphin to create awareness to save endangered species.
3. Scientists have created tiny spheres that can catch and destroy bisphenol A (BPA), recently in news. Consider the following about BPA. 1. It is a natural chemical used to make plastics that often contaminates water. 2. It can leach into water from the protective internal epoxy resin coatings of water bottles. 3. Microwaving poly-carbonate food containers helps get rid of BPA. Select the correct answer using the codes below.
    A. 1 only B. 2 and 3 only C. 1 and 3 only D. 1 and 2 only

Answer - Option D
Explanation:
Justification: BPA is commonly used to coat the insides of food cans, bottle tops and water supply lines, and as once a component of baby bottles.
Polycarbonate plastics have many applications including use in some food and drink packaging, e.g., water and infant bottles, compact discs, impact resistant safety equipment, and medical devices.
Epoxy resins are used as lacquers to coat metal products such as food cans, bottle tops, and water supply pipes. Some dental sealants and composites may also contribute to BPA exposure.
S2: Bisphenol A can leach into food from the protective internal epoxy resin coatings of canned foods and from consumer products such as polycarbonate tableware, food storage containers, water bottles, and baby bottles. The degree to which BPA leaches from polycarbonate bottles into liquid may depend more on the temperature of the liquid or bottle, than the age of the container. BPA can also be found in breast milk.
While BPA that seeps into food and drink is considered safe in low doses, prolonged exposure is suspected of affecting the health of children and contributing to high blood pressure.
The primary source of exposure to BPA for most people is through the diet. While air, dust, and water are other possible sources of exposure, BPA in food and beverages accounts for the majority of daily human exposure.
Statement 3: Don’t microwave polycarbonate plastic food containers. Polycarbonate is strong and durable, but over time it may break down from over use at high temperatures.
4. Information Sharing Platform Gateway for South-East Asia Regulatory Network (SEARN) has been developed by and is related to which of the following issues primarily?
    A. SAARC and Health B. BIMSTEC and Marine Piracy C. ASEAN and International trade D. None of the above

Answer - Option D
Explanation:
Learning: Centre for Development of Advanced Computing, India, has launched SEARN. It will promote regulatory and health collaboration among the countries of the South-East Asia Region.
In the South-East Asia region in WHO, India is actively contributing & providing support for the SEARN to guarantee access to high-quality medical products.
The South East Asia Research Network (SEARN), based at the London School of Hygiene & Tropical Medicine, is a platform to facilitate research collaboration.
It provides a forum to support the communication and dissemination of research findings, highlight research areas and a network connecting people and collaborators outside with an interest in South East Asia.
Composition:
SEARN includes all ASEAN countries: Thailand, Vietnam, Laos, Myanmar (Burma), Cambodia, Malaysia, Indonesia, Brunei, Singapore, Timor-Leste (East Timor) and the Philippines.
Objectives:
Encourage communication and dissemination of research findings.
Showcase ongoing research.
Act as a news forum
5. Consider the following statements about Azad Hind Government: 1. It was a provisional government formed by Netaji Subash Chandra Bose in Singapore. 2. The provisional government was supported by Allied powers during W.W.II. Which of the above statements is/are correct:
    A. 1 Only B. 2 Only C. Both 1 and 2 D. None

Answer - Option A
Explanation:
About Azad Hind Government:
Netaji Subhash Chandra Bose had announced the establishment of the provisional government of Azad Hind in occupied Singapore in 1943.
Known as Arzi Hukumat-e-Azad Hind, it was supported by the Axis powers of Imperial Japan, Nazi Germany, the Italian Social Republic, and their allies.
The revered freedom fighter had launched a struggle to free India from British rule under the banner of the provisional government-in exile during the latter part of the Second World War.
The formation:
On October 21, 1943, in his address announcing the setting up of the provisional government, he said India’s Army of Liberation was drawn up in military formation on the battlefield of Singapore which was once the bulwark of the British Empire.
Under the provisional government, Bose was the head of the state, the prime minister and the minister for war and foreign affairs.
Captain Lakshmi headed the women’s organisation while headed the publicity and propaganda wing.
Revolutionary leader Rash Behari Bose was designated as the supreme advisor. The provisional government was also formed in the Japanese-occupied Andaman and Nicobar Islands. The islands were reoccupied by the British in 1945.
Bose’s death was seen as the end to the Azad Hind movement. The Second World War, also ended in 1945 with the defeat of the Axis powers.
6. Recently first Regional Conference on ‘Women in Detention and Access to Justice’ was organized by Ministry of Home Affairs in which of the following state:
    A. Madhya Pradesh B. Gujarat C. Himachal Pradesh D. Haryana

Answer - Option C
Explanation: The Bureau of Police Research and Development (BPR&D), Ministry of Home Affairs is organising the First ever Regional Conference at Shimla on ‘Women in Detention and Access to Justice’ in collaboration with the Prison Department, Himachal Pradesh.
7. Justice Verma Committee often seen in news is related to:
    A. Empowerment of Schedule Tribes. B. Promoting National Integration. C. Resolving challenges faced by Minorities. D. Recommending reformed Gender Laws to address Sexual Harassment.

Answer - Option D
Explanation:
The Committee, chaired by Justice Verma and including Justice Leila Seth and senior lawyer Gopal Subramanium, termed the Sexual Harassment Bill “unsatisfactory” and said it did not reflect the spirit of the Vishakha guidelines — framed by the Supreme Court in 1997 to curb sexual harassment at the workplace. Major recommendations made by the panel:
• Punishment for Rape: The panel has not recommended the death penalty for rapists. It suggests that the punishment for rape should be rigorous imprisonment or RI for seven years.
• Punishment for other sexual offences: The panel recognised the need to curb all forms of sexual offences and recommended – Voyeurism be punished with upto seven years in jail; stalking or attempts to contact a person repeatedly through any means by up to three years. Acid attacks would be punished by up to seven years of imprisonment; trafficking will be punished with RI for seven to ten years.
• Registering complaints and medical examination: Every complaint of rape must be registered by the police and civil society should perform its duty to report any case of rape coming to its knowledge. Any officer, who fails to register a case of rape reported to him, or attempts to abort its investigation, commits an offence which shall be punishable as prescribed.
Marriages to be registered: As a primary recommendation, all marriages in India (irrespective of the personal laws under which such marriages are solemnised) should mandatorily be registered in the presence of a magistrate. The magistrate will ensure that the marriage has been solemnised without any demand for dowry having been made and that it has taken place with the full and free consent of both partners.
• Amendments to the Code of Criminal Procedure: The panel observed, “The manner in which the rights of women can be recognised can only be manifested when they have full access to justice and when the rule of law can be upheld in their favour.” The proposed Criminal Law Amendment Act, 2012, should be modified, suggests the panel.
• Bill of Rights for women: A separate Bill of Rights for women that entitles a woman a life of dignity and security and will ensure that a woman shall have the right to have complete sexual autonomy including with respect to her relationships.
• Review of the Armed Forces Special Powers Act: The panel has observed that the “impunity of systematic sexual violence is being legitimized by the armed forces special powers act.” It has said there is an imminent need to review the continuance of AFSPA in areas as soon as possible. It has also recommended posting special commissioners for women’s safety in conflict areas.
8. Consider the following statements about National Commission for Women: 1. It is a statutory body setup in 1992. 2. Commission publishes a monthly newsletter, Rashtra Mahila in Hindi and English Which of the above statements is/are correct:
    A. 1 Only B. 2 Only C. Both 1 and 2 D. None

Answer - Option C
Explanation:
The primary mandate of the National Commission for Women and State Commissions for Women Commission is to safeguard and protect the interests of women. It has wide responsibility covering almost all aspects of women development.
9. Recently Mahatma Gandhi International Sanitation Convention was held in which city of India?
    A. Varanasi B. Meerut C. New Delhi D. Gandhinagar

Answer - Option C
Explanation:
Mahatma Gandhi International Sanitation Convention:
The Mahatma Gandhi International Sanitation Convention is being organised by the Ministry of Drinking Water and Sanitation to mark the beginning of the 150th birth anniversary celebrations of Mahatma Gandhi, also coinciding with the fourth anniversary of the launch of Swachh Bharat Mission.
Ministers from over 70 countries will be invited and taken on a ‘Gandhi Trail’ in Gujarat.
The government will use the occasion to “showcase its performance” and “success story” in the Swachh Bharat programme in the past four years, which was launched on October 2, 2014, and have a face-to-face dialogue with the world leaders to share their experiences on sanitation programmes.
10. Consider the following statements about World Habitat Day: 1. UN designated First Monday of October of every year to observe the Day. 2. The purpose of the day is to reflect on the state of our cities and towns. Which of the above statements is/are correct:
    A. 1 Only B. 2 Only C. Both 1 and 2 D. None

Answer - Option C
Explanation:
The United Nations designated the first Monday of October of every year as World Habitat Day to reflect on the state of our towns and cities, and on the basic right of all to adequate shelter. The Day is also intended to remind the world that we all have the power and the responsibility to shape the future of our cities and towns.
• 2018 theme: Municipal Solid Waste Management. The focus of this year’s World Habitat Day celebrations is taking action to address the municipal solid waste management challenge. This year’s theme is Municipal Solid Waste Management with a slogan Waste-wise cities
1. Which of the following statements about Global Hunger Index is INCORRECT:
    A. Report is a peer-reviewed publication released annually by Welthungerhilfe and Concern Worldwide. B. GHI scores are based on three dimensions of hunger—insufficient caloric intake, child under nutrition, and child mortality. C. India has been ranked 57th in the GHI 2018. D. All are Correct.

Answer - Option C
Explanation:
The GHI scores are based on a formula that captures three dimensions of hunger—insufficient caloric intake, child under nutrition, and child mortality—using four component indicators:
UNDERNOURISHMENT: the share of the population that is undernourished, reflecting insufficient caloric intake
CHILD WASTING: the share of children under the age of five who are wasted (low weight-for-height), reflecting acute under nutrition.
CHILD STUNTING: the share of children under the age of five who are stunted (low height-for-age), reflecting chronic under nutrition.
CHILD MORTALITY: the mortality rate of children under the age of five.
Performance of India:
India has been ranked at 103 out of 119 countries in the Global Hunger Index 2018, with hunger levels in the country categorized as “serious”.
At least one in five Indian children under the age of five is wasted. The only country with a higher prevalence of child wasting is the war-torn nation of South Sudan.
2. Mount Etna is the volcano with longest record of continuous eruptions in the World. In which Continent is it?
    A. Africa B. South America C. Asia D. Europe

Answer - Option D
Explanation:
About Mt. Etna:
Mount Etna is the largest active volcano in Europe and one of the world’s most frequently erupting volcanoes. It is also the volcano with the longest record of continuous eruption.
Located near the east coast of the island of Sicily in Italy, Mount Etna is 10,900 feet (3,329 meters) tall with a base circumference of about 93 miles (150 kilometers).
Etna sits on the active fault between the African plate and the Ionian micro plate, which are both being subducted together beneath the Eurasian plate.
In June 2013, it was added to the list of UNESCO World Heritage Sites.
Due to its history of recent activity and nearby population, Mount Etna has been designated a Decade Volcano by the United Nations.
3. Recently, a term ‘Third Pole’ was seen in news. It denotes:
    A. Magnetic North Pole of Earth. B. Siberia Region of Russia C. Nordic Countries. D. Hindu Kush-Himalayan Range

Answer - Option D
Explanation:
The Hindu Kush-Himalayan region spans an area of more than 4.3 million square kilometres in Afghanistan, Bangladesh, Bhutan, China, India, Myanmar, Nepal, and Pakistan.
The region stores more snow and ice than anywhere else in the world outside the polar regions, giving its name: ’The Third Pole‘.
The Third Pole contains the world’s highest mountains, including all 14 peaks above 8,000 metres, is the source of 10 major rivers, and forms a formidable global ecological buffer.
Scientists conducting research in the third pole area have warned of disturbing global warming trends, and how, if they continue, they could affect the lives of 1.3 billion people.
The glacier has lost 60% of its mass and shrunk 250 m since 1982.
4. Consider the following statements about Environment Pollution (Prevention and Control) Authority: 1. It is a statutory body under MoE&CC. 2. The mandate of the body is to improve quality of environment in NCR Delhi. Which of the above statements is/are correct:
    A. 1 Only B. 2 Only C. Both 1 and 2 D. None

Answer - Option C
Explanation:
EPCA is Supreme Court mandated body tasked with taking various measures to tackle air pollution in the National Capital Region.
It was notified in 1998 by Environment Ministry under Environment Protection Act, 1986.
Besides the chairman, the EPCA has 14 members, some of whom are the environment secretary of the National Capital Territory of Delhi (NCT), chairperson of the New Delhi Municipal Council, transport commissioner of the NCT, the commissioners of various municipal corporations of Delhi and professors at IIT Delhi and Jawaharlal Nehru University.
Functions:
To protect and improve quality of environment and prevent and control environmental pollution in National Capital Region.
To enforce Graded Response Action Plan (GRAP) in NCR as per the pollution levels.
5. Consider the following statements about Indian Council of Forestry Research and Education: 1. It is a subsidiary body attached to MoEF & CC. 2. It promotes research in forestry including education and extension services in sustainable management of Forests. Which of the above statements is/are correct:
    A. 1 Only B. 2 Only C. Both 1 and 2 D. None

Answer - Option B
Explanation:
The Indian Council of Forestry Research and Education (ICFRE) is an autonomous council under the Union Environment Ministry.
The council mainly guides, promotes and coordinates forestry research, extension and education at the national level through its nine institutes and five centres located across the country.
The Indian Council of Forestry Research and Education (ICFRE) has signed two Memorandums of Understanding (MoUs) with Navodaya Vidyalaya Samiti (NVS) and Kendriya Vidyalaya Sangathan (KVS).
The agreements were signed to launch ‘Prakriti’ programme, which aims to promote awareness about forests and environment and stimulate interest among the students of NVS and KVS in maintaining a balanced environment and for acquiring skills that reflect care and protection towards forests, environment and society.
6. ‘Experiencing North East’ is a festival to showcase Diverse Culture and Heritage of NE India. It is organized by:
    A. Ministry of Culture B. Ministry of Tourism C. Ministry of Development of NE Region D. Ministry of National Integration

Answer - Option C
Explanation:
‘Experiencing North East’ Festival:
The event which is a part of ‘Destination North East’ series aims at showcasing the diverse culture and heritage of North East all under one roof.
The festival is being organised by North Eastern Council (NEC), Ministry of Development of North Eastern Region (DoNER) in collaboration with India International Centre.
The festival will showcase the vibrant strengths of North East India and display its art, handicraft, handlooms, tourism, food, culture etc.
Destination North East event is one of the steps taken by Ministry of Development of North Eastern Region to bring North East closer and help assimilate the diverse cultural heritage of India while encouraging investment in priority sectors like Tourism, Handloom, Handicrafts, Food Processing etc.
7. Consider the following statements about National Consumer Disputes Redressal Commission: 1. It is an executive body under the Ministry of Consumer Affairs. 2. The Commission acts as Quasi-Judicial Body at National Level relating to Consumer Disputes. Which of the above statements is/are correct:
    A. 1 Only B. 2 Only C. Both 1 and 2 D. None

Answer - Option B
Explanation:
About NCDRC:
The National Consumer Disputes Redressal Commission (NCDRC), India is a quasi-judicial commission in India which was set up in 1988 under the Consumer Protection Act of 1986.
The commission is headed by a sitting or retired judge of the Supreme Court of India.
Statutory provisions:
Section 21 of Consumer Protection Act, 1986 posits that the National Consumer shall have jurisdiction to entertain a complaint valued more than one crore and also have Appellate and Revisional jurisdiction from the orders of State Commissions or the District fora as the case may be.
Section 23 of Consumer Protection Act, 1986, provides that any person aggrieved by an order of NCDRC, may prefer an Appeal against such order to Supreme Court of India within a period of 30 days.
8. Consider the following statements Central Vigilance Commission: 1. It was originally setup as an executive body based on the recommendations of Santhanam Committee. 2. The Commission consists of a Central Vigilance Commissioner along with 2 vigilance commissioners. 3. The term of the members is 5 Years or 65 Years, whichever is earlier. Which of the above statements is/are correct:
    A. 1 and 2 Only B. 2 and 3 Only C. 1 and 3 Only D. All are Correct

Answer - Option A
Explanation:
It is the apex vigilance institution. It was created via executive resolution (based on the recommendations of Santhanam committee) in 1964 but was conferred with statutory status in 2003.
It submits its report to the President of India.
Composition: Presently, the body consists of central vigilance commissioner along with 2 vigilance commissioners.
Appointment: They are appointed by the President of India on the recommendations of a committee consisting of Prime Minister, Union Home Minister and Leader of the Opposition in Lok Sabha (if there is no LoP then the leader of the single largest Opposition party in the Lok Sabha).
Term: Their term is 4 years or 65 years, whichever is earlier.
Removal: The Central Vigilance Commissioner or any Vigilance Commissioner can be removed from his office only by order of the President on the ground of proved misbehavior or incapacity after the Supreme Court, on a reference made to it by the President, has, on inquiry, reported that the Central Vigilance Commissioner or any Vigilance Commissioner, as the case may be, ought to be removed.
9. Consider the following statements about Draft National Policy on Electronics 2018: 1. It aims to create $400 billion electronics manufacturing industry by 2025 2. It aims to double the production of Mobile Handsets to 1 Billion by 2025. Which of the above statements is/are correct:
    A. 1 Only B. 2 Only C. Both 1 and 2 D. None

Answer - Option C
Explanation:
Create $400 billion electronics manufacturing industry by 2025, with mobile phone devices segment accounting for three fourths of production. It also includes targeted production of 1 billion mobile handsets by 2025, valued at $190 billion (approximately Rs. 13 lakh crore) and also 600 million mobile handsets valued at $110 billion (approximately Rs. 7 lakh crore) for export.
Incentives: It replaces existing incentive schemes like Modified Special Incentive Package Scheme (M-SIPS), with schemes that are easier to implement such as interest subsidy and credit default guarantee etc.
It also proposes to push development of core competencies in all sub-sectors of electronics including electronic components and semiconductors, automotive electronics, defence electronics, industrial electronics, strategic electronics etc.
It also proposes to set up 20 greenfield and three brownfield electronic manufacturing cluster projects have been sanctioned with project outlay of Rs 3,898 crore, including Rs 1,577 crore from Central Government.
10. Which of the following states is set to become smoke free state due to high penetration of LPG:
    A. Karnataka B. Tamil Nadu C. Gujarat D. Kerala

Answer - Option D
Explanation:
Kerala is now set to become the first smoke-free State in the country with public sector oil companies eyeing 100% LPG penetration here.
Kerala is one of the States with the highest penetration of LPG, which is transforming lifestyles. The target has almost been achieved in most villages, towns and cities in the Kerala.
LPG consumption was 933.3 TMT (thousand tonnes) in the Kerala in 2017-18. It is estimated that one crore tonnes of emissions, from poisonous gases like firewood cooking, have been contained and 25 lakh trees have been saved.
1. Consider the following statements about Mega Food Parks Scheme: 1. Ministry of Rural Development is the implementing authority for the scheme. 2. Government makes a contribution of 50% of the project cost with maximum grant of Rs 100 Crore. Which of the above statements is/are correct:
    A. 1 Only B. 2 Only C. Both 1 and 2 D. None

Answer - Option D
Explanation:
About Mega Food Parks:
Ministry of Food Processing Industries is implementing Mega Food Park Scheme in the country.
The Scheme of Mega Food Park aims at providing a mechanism to link agricultural production to the market by bringing together farmers, processors and retailers so as to ensure maximizing value addition, minimizing wastages, increasing farmers’ income and creating employment opportunities particularly in rural sector.
These food parks give a major boost to the food processing sector by adding value and reducing food wastage at each stage of the supply chain with particular focus on perishables.
A maximum grant of Rs 50 crore is given for setting up a MFP, in minimum 50 acres of contiguous land with only 50% contribution to the total project cost.
Mode of operation:
The Scheme has a cluster based approach based on a hub and spokes model. It includes creation of infrastructure for primary processing and storage near the farm in the form of Primary Processing Centres (PPCs) and Collection Centres (CCs) and common facilities and enabling infrastructure at Central Processing Centre (CPC).
The PPCs are meant for functioning as a link between the producers and processors for supply of raw material to the Central Processing Centres.
CPC has need based core processing facilities and basic enabling infrastructure to be used by the food processing units setup at the CPC. The minimum area required for a CPC is 50 acres.
The scheme is demand-driven and would facilitate food processing units to meet environmental, safety and social standards.
2. Consider the following statements about Rashtriya Vayoshri Yojana: 1. It is a Centrally Sponsored Scheme implemented by Ministry of Social Justice and Empowerment. 2. The Scheme aims at providing Senior Citizens, belonging to BPL category and suffering from any of the age-related disability/infirmity with assisted-living devices which can restore near normalcy in their bodily functions. Which of the above statements is/are correct:
    A. 1 Only B. 2 Only C. Both 1 and 2 D. None

Answer - Option B
Explanation:
This is a Central Sector Scheme, fully funded by the Central Government.
The expenditure for implementation of the scheme will be met from the “Senior Citizens’ Welfare Fund“.
Under the scheme, free of cost distribution of the devices, commensurate with the extent of disability/infirmity that is manifested among the eligible senior citizens will take place.
In case of multiple disabilities/infirmities manifested in the same person, the assistive devices will be given in respect of each disability/impairment.
Beneficiaries in each district will be identified by the State Governments/UT Administrations through a Committee chaired by the Deputy Commissioner/District Collector.
3. Consider the following statements about Saubhagya Scheme: 1. Rural Electrification Corporation is the nodal agency for the scheme. 2. It aims to provide free electricity connections to BPL households. Which of the above statements is/are correct:
    A. 1 Only B. 2 Only C. Both 1 and 2 D. None

Answer - Option A
4. Consider the following statements about Pradhan Mantri Bhartiya Janaushadhi Pariyojana: 1. It aims at providing quality medicines at affordable prices to the masses through special Kendras. 2. The campaign is run by Department of Pharmaceuticals, Government of India Which of the above statements is/are correct:
    A. 1 Only B. 2 Only C. Both 1 and 2 D. None

Answer - Option C
Explanation:
‘Pradhan Mantri Bhartiya Janaushadhi Pariyojana’ is a campaign launched by the Department of Pharmaceuticals, Govt. Of India, to provide quality medicines at affordable prices to the masses through special kendra’s known as Pradhan Mantri Bhartiya Jan Aushadhi Kendra.
Pradhan Mantri Bhartiya Jan Aushadhi Kendra (PMBJK) have been set up to provide generic drugs, which are available at lesser prices but are equivalent in quality and efficacy as expensive branded drugs.
Bureau of Pharma PSUs of India (BPPI) is the implementing agency of PMBJP. BPPI (Bureau of Pharma Public Sector Undertakings of India) has been established under the Department of Pharmaceuticals, Govt. of India, with the support of all the CPSUs.
Central Warehouse of Pradhan Mantri Bhartiya Janaushadhi Pariyojana has been set up at Bilaspur, Gurugram. It has been setup by the Bureau of Pharma PSUs of India (BPPI).
The hi-tech Central Warehouse would facilitate seamless distribution of Jan Aushadhi generic medicines to all PMBJP Kendra functional across the country.
5. Asian Development Bank and Government of India have recently signed a $150 Million loan agreement to setup a Global Skills Park. It will be setup in:
    A. Rajasthan B. Uttar Pradesh C. Jharkhand D. Madhya Pradesh

Answer - Option D
Explanation:
About the Global Skills Park (GSP):
It will be the First Multi-Skills Park in India, to enhance the quality of Technical and Vocational Education And Training (TVET) System in the State and create a more skilled workforce.
The Project will engage international TVET partners to support advanced training at the GSP who will bring global best practices in TVET management, training infrastructure, industry cooperation, and quality assurance.
The GSP campus will consist of core Advanced Training Institutes including the Center for Occupational Skills Acquisition and the Center for Advanced Agricultural Training as well as other support services focusing on entrepreneurship, training of trainers, and skill-related research.
The campus will have training facilities focusing on skills for manufacturing, service, and advanced agricultural jobs, benefiting about 20,000 trainees and trainers.
6. Closing the Skills Gap Project recently seen in news is a project of:
    A. World Bank B. BRICS C. United Nations Development Programme D. World Economic Forum

Answer - Option D
Explanation:
Closing the Skills Gap Project by WEF:
The Closing the Skills Gap Project aims to create global and national platforms to address current skills gaps and to reshape education and training for the future.
The government has launched a task force for closing the skills gap in India, in collaboration with the World Economic Forum.
The Task Force is the second country-led public-private collaboration of the World Economic Forum’s Closing the Skills Gap Project after South Africa.
7. After Assam, Which of the following states is proposed to come up with National Register of Citizens:
    A. Meghalaya B. West Bengal C. Manipur D. Tripura

Answer - Option D
Explanation:
The Supreme Court has issued notice to the government to update the National Register of Citizens (NRC) in Tripura, as is being done in Assam, in order to detect and deport the “illegal immigrants” from Bangladesh.
8. The government has set up Strategic Policy Group (SPG). Consider the following about it. 1. It will advise the National Security Council, headed by the President of India, on matters of national security and strategic interests. 2. The Cabinet secretary, the chiefs of the three defence services, the RBI governor, and the foreign secretary, inter alia, form part of the group. Which of the above is/are correct?
    A. 1 Only B. 2 Only C. Both 1 and 2 D. None

Answer - Option B
Explanation:
Justification: The NSC is headed by the Prime Minister.
SPG will be headed by National Security Advisor. Its members include the NITI Aayog vice chairman, Cabinet Secretary, the chiefs of the three defence services, the RBI governor, the foreign secretary, the home secretary, the finance secretary and the defence secretary.
The Secretary of the Department of Defence Production and Supplies, the Scientific Adviser to the Defence Minister and the Secretary, Cabinet Secretariat will also be members of the panel.
The other members are secretary, department of revenue; secretary, department of atomic energy; secretary, department of space; director, Intelligence Bureau, and secretary, National Security Council Secretariat.
Representatives of other ministries and departments will be invited to the meetings of the group as and when necessary.
It will assist the National Security Council and undertake among other tasks, a long-term strategic review of country’s security affairs.
It will be the principal mechanism for inter-ministerial coordination and integration of relevant inputs in the formulation of national security policies.
9. Consider the following statements: 1. Recently, the Union Cabinet has merged National Council for Vocational Training and National Skill Development Agency into National Council for Vocational Education and Training. 2. The resultant body will regulate the various entities engaged in Vocational Education and Training. Which of the above statements is/are correct:
    A. 1 Only B. 2 Only C. Both 1 and 2 D. None

Answer - Option C
10. Human Capital Index is released by:
    A. World Bank B. World Economic Forum C. United Nations Development Programme. D. International Monetary Fund

Answer - Option A
The World Bank has released a Human Capital Index (HCI) as part of the World Development Report 2019.
Theme for the World Development Report (WDR) 2018: “The Changing Nature of Work”.
About Human Capital Index (HCI):
The HCI has been constructed for 157 countries. It claims to seek to measure the amount of human capital that a child born today can expect to attain by age 18.
The HCI index values are contended to convey the productivity of the next generation of workers, compared to a benchmark of complete standard education and full health.
The HCI has three components:
Survival, as measured by under-5 mortality rates.
Expected years of Quality-Adjusted School which combines information on the quantity and quality of education.
Health environment using two proxies of (a) adult survival rates and (b) the rate of stunting for children under age 5.
HCI Vs. HDI:
UNDP constructs Human Development Index (HDI) for several years.
The HCI uses survival rates and stunting rate instead of life expectancy as measure of health, and quality-adjusted learning instead of merely years of schooling as measure of education.
HCI also excludes per capita income whereas the HDI uses it. Two significant changes from HDI are exclusion of income component and introduction of quality adjustment in learning.
Exclusion of income element and introduction of quality adjustment makes HCI far less representative of Human Capital Development than the Index claims it to be.
1. Consider the following statements about Global Competitiveness Index 2018: 1. The Index is released by World Economic Forum. 2. India’s Rank was 58th in the recent release. Which of the above statements is/are correct:
    A. 1 Only B. 2 Only C. Both 1 and 2 D. None

Answer - Option C
Explanation:
India was ranked as the 58th most competitive economy with a score of 62.0 on the Global Competitiveness Index 2018.
India jumped five spots from 2017, the largest gain among G20 economies.
India ranked highest among South Asian countries. Sri Lanka was ranked 86th, Bangladesh 103rd, Pakistan 107th and Nepal 109th.
As per the report, India leads the region in all other areas of competitiveness except for health, education and skills.
As per the report, India’s greatest competitive advantages include its market size and innovation.
On the list of 140 economies, the United States topped the list with a score of 85.6, followed by Singapore and Germany at the second and the third positions respectively.
Among the BRICS economies, China topped the list at 28th place with a score of 72.6, followed by Russia, India, South Africa and Brazil respectively.
Background:
The Global Competitiveness Index (GCI) is prepared on the basis of country-level data covering 12 categories or pillars of competitiveness.
Institutions, infrastructure, macroeconomic environment, health and primary education, higher education and training, goods market efficiency, labour market efficiency, financial market development, technological readiness, market size, business sophistication and innovation are the 12 pillars.
2. Recently, Union Government launched SPARC scheme. It is associated with:
    A. Enhancing Energy Efficiency in Industry B. Promoting academic Research Collaboration with foreign research groups. C. Addressing the grievances of Labour Force in unorganized sector. D. None of the Above.

Answer - Option B
Explanation:
Ministry of Human Resource Development has launched the web portal of the Scheme “Scheme for Promotion of Academic and Research Collaboration (SPARC)”.
About Scheme for Promotion of Academic and Research
Collaboration (SPARC):
The Government in August 2018 had sanctioned the scheme “Scheme for Promotion of Academic and Research Collaboration (SPARC)” at a total cost of Rs.418 Cr for implementation up to 31.3.2020.
Indian Institute of Technology Kharagpur is the National Coordinating Institute to implement the SPARC programme. Details may be viewed at www.sparc.iitkgp.ac.in.
The scheme aims at improving the research ecosystem of India’s higher educational institutions by facilitating academic and research collaborations between Indian Institutions and the best institutions in the world.
Under this Scheme, 600 joint research proposals will be awarded for 2 years to facilitate strong research collaboration between Indian research groups with the best in class faculty and renowned research groups in the leading universities of the world, in areas that are at the cutting edge of science or with direct social relevance to the humankind, specifically India.
3. Recently, Swachh Survekshan Awards 2018 were given. Which of the following states topped as the best state?
    A. Kerala B. Haryana C. Meghalaya D. Gujarat

Answer - Option B
Explanation:
Prime Minister recently conferred Swachh Survekshan Grameen Awards 2018 to top ranked states and districts. The rankings were based on the National Swachh Survekshan Grameen 2018 of the Union Ministry of Drinking Water and Sanitation.
Haryana was ranked as best State.
Satara District of Maharashtra was ranked as best district.
Uttar Pradesh was rewarded for maximum citizens’ participation.
About Swachh Survekshan Grameen:
SSG is a rural cleanliness survey to rank all states and districts on basis of qualitative and quantitative evaluation.
The objective of SSG 2018 is to undertake ranking of states and districts on basis of their performance attained on key quantitative and qualitative Swachh Bharat Mission-Grameen (SBM-G) parameters. The rankings will be based on taking into account set of comprehensive cleanliness parameters.
The criteria of SSG-2018 include survey of public places, citizens’ perspective of cleanliness, their recommendations and data from SBM-G. As part of it, more than 6000 villages in 698 districts across India were covered. It covered nearly 30,000 public places namely schools, anganwadis, public health centres, haat/bazaars/religious places in these villages
4. BepiColombo, recently in the news, is a joint mission between
    A. NASA and ESA B. ESA and JAXA C. JAXA and NASA D. ESA and ROSCOSMOS

Answer - Option B
Explanation:
BepiColombo is Europe’s first mission to Mercury.
It has set off recently on a journey to the smallest and least explored terrestrial planet in our Solar System. When it arrives at Mercury in late 2025, it will endure temperatures in excess of 350 °C and gather data during its 1 year nominal mission, with a possible 1-year extension. BepiColombo is a joint mission between ESA and the Japan Aerospace Exploration Agency (JAXA), executed under ESA leadership.
5. With reference to the ‘International Civil Aviation Organisation’, which of the following statements is/are correct? 1. It is a United Nations Specialized agency. 2. It manages the administration and governance of the Chicago Convention. Select the correct answer using the code given below:
    A. 1 only B. 2 only C. Both 1 and 2 D. Neither 1 nor 2

Answer - Option C
Explanation:
About ICAO: ICAO’s headquarters is located in Montreal, Canada.
The ICAO is a UN specialized agency, established by States in 1944 to manage the administration and governance of the Convention on International Civil Aviation (Chicago Convention).
ICAO works with the Convention’s 191 Member States and industry groups to reach consensus on international civil aviation Standards and Recommended Practices (SARPs) and policies in support of a safe, efficient, secure, economically sustainable and environmentally responsible civil aviation sector. These SARPs and policies are used by ICAO Member States to ensure that their local civil aviation operations and regulations conform to global norms.
ICAO also coordinates assistance and capacity building for States in support of numerous aviation development objectives and audits States’ civil aviation oversight capabilities in the areas of safety and security.
In the news (TH): The Air Passengers Association of India (APAI) has expressed concern over India’s low ‘air safety oversight score’, which is lower than that of Myanmar, Bangladesh, Maldives, Pakistan, Sri Lanka, Nepal and North Korea in the Asia-Pacific region. The ICAO Universal Safety Oversight Audit Programme seeks to identify if countries have consistently implemented a safety-oversight system. India is one of the 15 countries that are below the minimum target rates.
6. To “ensure availability and sustainable management of water and sanitation for all” is Sustainable Development Goal #6. Some targets under it include 1. By 2020, protect and restore water-related ecosystems including mountains, forests, wetlands, rivers aquifers and lakes. 2. By 2025, end open defecation. 3. By 2030, substantially increase water-use efficiency across all sectors and substantially reduce the number of people suffering from water scarcity. Select the correct answer using the code given below:
    A. 1 and 2 only B. 2 and 3 only C. 1 and 3 only D. 1, 2 and 3

Answer - Option
Explanation:
Among others, targets under SDG #6 include:
By 2030, achieve universal and equitable access to safe and affordable drinking water for all.
By 2030, achieve access to adequate and equitable sanitation and hygiene for all and end open defecation.
By 2030, improve water quality by reducing pollution and substantially increasing recycling and safe reuse globally.
By 2030, substantially increase water-use efficiency across all sectors and ensure sustainable withdrawals and supply of freshwater to address water scarcity.
By 2030, implement integrated water resources management at all levels.
By 2020, protect and restore water-related ecosystems, including mountains, forests, wetlands, rivers, aquifers and lakes.
7. The “Moscow format”, and “Tashkent Declaration”, sometimes in the news, both appear in the context of affairs related to
    A. Denuclearisation B. International North-South Transport Corridor C. Turkistan Islamic Movement D. Taliban

Answer - Option D
Explanation:
On Afghanistan, India expressed support for the “Moscow format”, in which Russia involves regional countries and major powers in an effort to draw the Taliban into negotiations with the Afghan leadership. The U.S. has boycotted this initiative, but has initiated its own dialogue with the Taliban.
In a joint statement at the end of official discussions with the visiting Uzbek leader, both sides pledged to work on the goals of the Tashkent Declaration. The Tashkent Declaration had called for “direct negotiation with the Taliban without any preconditions.”
8. The Intermediate-Range Nuclear Forces Treaty (INF Treaty), is a nuclear arms-control accord that was reached in 1987 by
    A. USA and Russia only B. NPT-designated nuclear states only, except China C. All countries party to the NPT treaty D. Members of the NSG only

Answer - Option A
Explanation:
Intermediate-Range Nuclear Forces Treaty (INF Treaty), is a nuclear arms-control accord reached by the United States and the Soviet Union in 1987 in which those two nations agreed to eliminate their stocks of intermediate-range and shorter-range (or “medium-range”) land-based missiles (which could carry nuclear warheads). It was the first arms-control treaty to abolish an entire category of weapon systems. In addition, two protocols to the treaty established unprecedented procedures for observers from both nations to verify firsthand the other nation’s destruction of its missiles.
The INF Treaty defined intermediate-range ballistic missiles (IRBMs) and ground-launched cruise missiles (GLCMs) as those having ranges of 1,000 to 5,500 km (620 to 3,400 miles) and shorter-range ballistic missiles (SRBMs) as those having ranges from 500 to 1,000 km.
In the news: USA announces withdrawal from INF treaty;
9. ‘Jewel of Roman Empire’, the world heritage site and an ancient city of Sabratha is located in which country?
    A. Iran B. Yemen C. Libya D. Moscow

Answer - Option C
10. The Genome Asia 100 K initiative will 1. Help accelerate medical advances and precision medicine for the Indian population. 2. Make available the data collected for the public. 3. Sequence the genes of 50,000 Indians. Which of the statements given above is/are correct ?
    A. 1 and 2 only B. 2 and 3 only C. 1 and 3 only D. 1, 2 and 3

Answer - Option D
Explanation:
About Genome Asia 100 K
“A mission driven non-profit consortium collaborating to sequence and analyze 100,000 Asian individuals genomes to help accelerate Asian (all Asia, including India) population specific medical advances and precision medicine. Through an unprecedented commitment to open information, the data from Genome Asia 100 K will be made available to the public.”
A group of Indian scientists and companies are involved with a 100 k Genome Asia project, led out of the Nanyang Technological University (NTU), Singapore, to sequence the whole genomes of 100 k Asians, including 50,000 Indians.
1. CAATSA was in news recently due to which of the following issues?
    A. Imposing sanctions on Iran, Iraq and Afghanistan B. Warning sanctions under the India-Russsia deal of S-400 ‘Triumf’ missile systems C. Both (a) and (b) D. None

Answer - Option B
Explanation:
Justification and Learning: CAATSA is a US federal law that imposed sanctions on Iran, North Korea and Russia. It includes sanctions against countries that engage in significant transactions with Russia’s defence and intelligence sectors.
However, any imposition of sanctions on India, which is now a major defence partner, could be disastrous for the bilateral relationship.
India and Russia have concluded the contract for five S-400 ‘Triumf’ missile systems, one of the biggest defence deals in recent times.
However, the U.S. has warned the deal would invoke sanctions under the Countering America’s Adversaries Through Sanctions Act (CAATSA) law.
What is S-400?
It is an air defence missile system that can take down enemies’ aircraft in the sky from the surface itself.
The S-400 is known as Russia’s most advanced long-range surfaceto-air missile defence system, capable of destroying destroying hostile strategic bombers, jets, missiles and drones at a range of 380-km.
The S-400 is an upgraded version of the S-300 systems. The missile system, manufactured by Almaz-Antey, has been in service in Russia since 2007.
2. Methanol is a promising fuel as it is clean, cheaper than fossil fuels and a good substitute for heavy fuels. Consider the following about it. 1. Methanol can be produced from renewable sources such as biomass and recycled carbon dioxide. 2. India is able to meet its methanol requirements domestically and exports it to Gulf and Central Asian nations. 3. Methanol burning does not cause sulphur oxides (SOx) emissions. Select the correct answer using the codes below.
    A. 1 and 2 only B. 1 and 3 only C. 3 only D. 1 only

Answer - Option D
Explanation:
Justification: India imports methanol from Saudi Arabia and Iran at present. Across the world, methanol is emerging as a clean, sustainable transportation fuel of the future.
This is because Methanol can be used as an energy producing fuel, transportation fuel and cooking fuel, cutting down India’s oil import bill by an estimated 20% over the next few years. Unlike CNG, using methanol as a transportation fuel would require minimal alteration in the vehicles.
S1 and S3: Methanol is a clean-burning fuel that produces fewer smog causing emissions — such as sulphur oxides (SOx), nitrogen oxides (NOx) and particulate matter — and can improve air quality and related human health issues.
Methanol is most commonly produced on a commercial scale from natural gas. It can also be produced from renewable sources such as biomass and recycled carbon dioxide.
As a high-octane vehicle fuel, methanol offers excellent acceleration and power. It also improves vehicle efficiency.
Methanol can be tapped back to produce Methanol. Thereby a seamless loop of CO2 sequestration cycle is created to perpetually burn fuels without polluting the environment at all. C02 from steel plants, Thermal Power plants, Cement Plants etc. can be tapped in large quantities to produce Methanol.
3. Recently, Assam became the first state to offer wage compensation scheme to pregnant women working in tea plantations in the State. In this context, consider the following statements: 1. Assam has a very high rate of maternal mortality especially in case of women working on tea plantations. 2. The State government proposes to provide Rs12000 to Pregnant women working on tea plantations in 4 installments. Which of the above statements is/are correct:
    A. 1 Only B. 2 Only C. Both 1 and 2 D. None

Answer - Option C
Explanation:
Assam Government has become the first Indian state to offer a Wage Compensation Scheme for pregnant women working in the tea gardens of the state.
Aim: The scheme is aimed at providing better health and nutrition supplements to the pregnant women. It stresses on providing proper healthcare facilities to the pregnant women working in the tea gardens of the state.
Under the scheme, an amount of Rs 12,000 will be given to the pregnant women so that they can take care of themselves and the unborn baby without compromising the livelihood of their family.
The compensation of wages to pregnant women will be given in 4 installments – Rs 2,000 in the first trimester, Rs 4,000 in the second trimester, Rs 3,000 for institutional delivery and Rs 3,000 for registration of the child’s birth.
The women would also be given a maternity leave. They will not be engaged in work from the third trimester of pregnancy to three months after delivery.
In addition, they will get assistance for ante-natal care and the first cycle of immunization of the child.
Background:
The maternal mortality rate of women working in the tea plantations of Assam is unusually high. In the Annual Health Survey of 2012-13, Assam recorded one of the highest maternal mortality rates in India, with over 300 maternal deaths per 100,000 live births. The MMR in the state’s tea gardens was even higher, as it was recorded to be 404. The national average during 2014-16 was 130.
Also, almost 50% of the pregnant women aged between 15 and 49 years in the state were recorded to be anaemic, which is a leading contributor to maternal mortality. The bulk of the workforce in Assam’s tea gardens is women.
4. Operation Samudra Maitri was launched by India to help:
    A. Nepal Earthquake Victims. B. Earthquake and Tsunami Victims of Indonesia. C. Cyclone Titli Victims D. Rescue Indians stranded in Oman

Answer - Option B
Explanation:
India has launched massive humanitarian operation Samudra Maitri to provide assistance to earthquake and tsunami victims in Indonesia.
Under this operation, India has dispatched two aircraft C-130J and C-17 and three naval ships carrying relief material and personnel to the country.
Operation launched to help Nepal Earthquake Victims (2015) was ‘Operation Maitri’
5. Which of the following statements about Government E Payments Adoption Ranking is incorrect:
    A. Ranking has been released by VISA, a global leader in payments technology B. It ranks governments by quantifying their e-payment capabilities. C. The ranking is based on 10 indicators. D. ndia was ranked 28th in 2018 Rankings.

Answer - Option C
Explanation:
The 2018 Government E-Payments Adoption Ranking (GEAR) study has been released by VISA, a global leader in payments technology. This is the third edition of the study after those in 2007 and 2011.
The 2018 GEAR, an Economist Intelligence Unit (EIU) global Index and bench marking study commissioned by Visa, ranks governments by quantifying their e-payment capabilities based on various indicators.
The ranking is based on seven parameters viz. Government-to-Citizen (G2C), Citizen-to-Government (C2G), Business-to-Government (B2G), Government-to-Business (G2B) transactions, infrastructure, socioeconomic and policy environment.
In the latest study, India is ranked 28th among 73 countries. This is up from 36th rank in 2011.
6. Recently Digi Yatra was in news in the context of promoting hassle-free and paper less travel for:
    A. Rail Travel B. Air Travel C. Inland Waterway Travel D. None of these

Answer - Option B
Explanation:
Union Ministry of Civil Aviation has released policy on biometric based digital processing of passengers at airports called Digi Yatra. With this initiative, ticket booking, airport entry and boarding pass security check-in will be made digital.
Digi Yatra:
The initiative seeks to promote paperless and hassle-free air travel. It will be operational by end of February, 2019 at Bengaluru and Hyderabad airports. In later phase, Airports Authority of India (AAI) will roll out this initiative at Kolkata, Varanasi, Pune and Vijayawada airports by April 2019.
Under it, there will be one-time verification at departure airport while travelling for first time using ID. After successful verification, facial recognition biometric will be captured and stored in Digi Yatra ID.
For this system, passengers will be registered through centralized system and will be given Digi Travel ID. This ID will include details such as names of passengers, their e-mail id, mobile number and any other identity card in case of non-basis. Travelers can also use this ID when booking tickets.
7. Med Watch is an innovative mobile health app which provides its users health information. It is launched by:
    A. Ministry of Electronics and Information Technology B. Ministry of Health and Family Welfare C. Ministry of Youth Affairs. D. Indian Air Force.

Answer - Option D
Explanation:
It is an innovative mobile health App launched by the Indian Air Force (IAF) to provide health information to the users, including first-aid and other health and nutritional topics. It is the first mobile health app in the three Armed Services.
Key facts:
The app is conceived by the doctors of IAF and developed in house by Directorate of Information Technology (DIT) with ZERO financial outlay.
8. Consider the following statements: 1. Sittwe is a port city of Myanmar located at the mouth of Kaladan River. 2. India is developing a multi modal transit transport project which aims to connect North Eastern India with the rest of India through Sittwe Port. Which of the above statements is/are correct:
    A. 1 Only B. 2 Only C. Both 1 and 2 D. None

Answer - Option C
Explanation:
India and Myanmar have signed an important MoU for the appointment of a private Port Operator for the Operation and Maintenance of Sittwe Port, Paletwa Inland Water Terminal and associated facilities included in the Kaladan Multi Model Transit Transport Project in implementation of India’s Act East Policy.
Where is Sittwe located?
Sittwe is the capital of Rakhine State (which has been in the news for the plight of Rohingya Muslims) in south-western Myanmar. It is located at the mouth of the Kaladan river, which flows into Mizoram in north-eastern India.
About Kaladan project:
The Kaladan project connects Sittwe Port in Myanmar to the IndiaMyanmar border.
The project was jointly initiated by India and Myanmar to create a multimodal platform for cargo shipments from the eastern ports to Myanmar and to the North-eastern parts of the country through Myanmar.
It is expected to open up sea routes and promote economic development in the North-eastern states, and also add value to the economic, commercial and strategic ties between India and Myanmar.
This project will reduce distance from Kolkata to Sittwe by approximately 1328 km and will reduce the need to transport good through the narrow Siliguri corridor, also known as Chicken’s Neck.
9. Consider the following statements. Assertion (A): A Member of the International Court of Justice (ICJ) is a delegate of the government of his own country. Reason (R): The ICJ is composed of representatives of governments as jurists. In the context of the above, which of these is correct?
    A. A is correct, and R is an appropriate explanation of A. B. A is correct, but R is not an appropriate explanation of A. C. A is correct, but R is incorrect. D. Both A and R are incorrect.

Answer - Option D
Explanation:
Justification: A Member of the ICJ is a delegate neither of the government of his own country nor of that of any other State.
Unlike most other organs of international organizations, the Court is not composed of representatives of governments.
The International Court of Justice is composed of 15 judges elected to nine-year terms of office by the United Nations General Assembly and the Security Council.
These organs vote simultaneously but separately. In order to be elected, a candidate must receive an absolute majority of the votes in both bodies.
In order to ensure a measure of continuity, one third of the Court is elected every three years. Judges are eligible for re-election.
Members of the Court are independent judges whose first task, before taking up their duties, is to make a solemn declaration in open court that they will exercise their powers impartially and conscientiously.
In order to guarantee his or her independence, no Member of the Court can be dismissed unless, in the unanimous opinion of the other Members, he/she no longer fulfills the required conditions. This has in fact never happened.
10. Which of these South Asian countries has abolished death penalty completely for all crimes? 1. Bhutan 2. Sri Lanka 3. Nepal 4. Bangladesh Select the correct answer using the codes below
    A. 1 and 2 only B. 1 and 3 only C. 1 only D. 2, 3 and 4 only

Answer - Option B
Explanation:
Justification: Bhutan abolished the death penalty for all crimes in 2004. Nepal did so for ordinary crimes in 1997 and full abolition later.
Recently, Malaysia also abolished it for all crimes.
1. Consider the following statements about GLOBAL REALTY TRANSPARENCY INDEX: 1. It is an index that measures country’s transparency in Real Estate sector. 2. It is released by World Bank. 3. India’s was ranked 35th in the recent release of Index. Which of the above statements is/are correct:
    A. 1 and 3 Only B. 2 and 3 Only C. 1 and 2 Only D. 2 Only

Answer - A
Explanation -
Jones Lang LaSalle Inc. (JLL) has released the Global Real Estate Transparency Index 2018.
The 2018 Global Real Estate Transparency Index covers 100 markets and is based on 186 indicators.
These variables are divided into six areas –performance measurement, market fundamentals, governance of listed vehicles, regulatory & legal frameworks, transaction process and environmental sustainability.
The Index scores markets on a scale of 1 to 5 (with 1.00 being the highest possible score). Depending on their overall performance, markets are assigned to one of five transparency tiers.
    1. Highly Transparent. 2. Semi-Transparent. 3. Low Transparency.

Performance of various countries:
India has moved up just one spot from 36 in 2016 to 35 in 2018.
The UK, Australia, the US, France and Canada are the top five countries.
Sri Lanka is at the 66th position and Pakistan at 75th among south Asian countries. Venezuela is the least transparent market with 100th rank.
Among BRICS nations, both China and South Africa remained on the same rank 33rd and 21st position, respectively, while, Brazil slipped to 37th position and Russia remained at 38th rank.
2. Consider the following statements regarding Unlawful Activities (Prevention) Act: 1. It is a legislative mechanism to deal with the activities that threaten the sovereignty and territorial integrity Of India. 2. It was framed in 1967 and has been amended since then for two times. Which of the above statements is/are correct:
    A. 1 Only B. 2 Only C. Both 1 and 2 D. None

Answer - C
Explanation - This law is aimed at effective prevention of unlawful activities and associations in India.
Its main objective is to make powers available for dealing with activities directed against the integrity and sovereignty of India.
The Act makes it a crime to support any secessionist movement or to support claims by a foreign power to what India claims as its territory.
The UAPA, framed in 1967, has been amended twice since: first in 2008 and then in 2012.
3. Consider the following statements about Global Environment Facility: 1. It is a mechanism to provide grants for environmental projects. 2. It was established in 2010 in line with the rising demand for funds for environmental projects. 3. World Bank act as the trustee to fund. Which of the above statements is/are correct:
    A. 1 and 2 B. 2 and 3 C. 1 and 3 D. All are correct

Answer - C
Explanation - The Global Environment Facility was established on the eve of the 1992 Rio Earth Summit to help tackle our planet’s most pressing environmental problems.
It is an international partnership of 183 countries, international institutions, civil society organizations and the private sector that addresses global environmental issues.
GEF funds are available to developing countries and countries with economies in transition to meet the objectives of the international environmental conventions and agreements.
The World Bank serves as the GEF Trustee, administering the GEF Trust Fund.
4. Consider the following statements in the context of E-Vehicles: 1. G.O.I have a dedicated scheme FAME for faster adoption and Manufacturing of E-Vehicles. 2. The scheme is currently implemented by Ministry of Heavy Industries. Which of the above statements is/are correct:
    A. 1 Only B. 2 Only C. 1 and 2 Only D. None

Answer - C
Explanation - With an aim to promote eco-friendly vehicles, the government had launched the Faster Adoption and Manufacturing of (Hybrid &) Electric Vehicles in India (FAME-India) scheme in 2015. It was launched by union ministry for heavy industries.
Aim: The FAME India Scheme is aimed at incentivizing all vehicle segments, including two-wheeler, three wheeler auto, passenger four-wheeler vehicle, light commercial vehicles, and buses. The scheme covers hybrid and electric technologies like a strong hybrid, plug-in hybrid and battery electric vehicles.
Facts: FAME India – Faster Adoption and Manufacturing of Hybrid and Electric vehicles in India – is a part of the National Electric Mobility Mission Plan. The scheme envisages Rs 795 crore support in the first two fiscals. It is being administered by the Heavy Industries Ministry.
The government seeks to withdraw the cash incentives for private electric cars because it neither makes a “substantial difference in promoting sales nor serves the purpose of a clean environment”.
Instead, the government has now decided to give cash subsidies to electric vehicles used by shared-mobility operators such as ola and Uber, as their vehicles will run much more than private cars.
5. Which of the following space agencies has launched “Remove Debris”; first spacecraft to remove debris orbiting sun:
    A. NASA B. ISRO C. European Space Agency (ESA) D. None of these

Answer - Option D
Explanation - Remove Debris is an EU (European Union) research project to develop and fly a low cost in-orbit demonstrator mission that aims to de-risk and verify technologies needed for future ADR (Active Debris Removal) missions.
6. GLONASS is native GPS version of which country
    A. Japan B. China C. India D. Russia

Answer - Option D
Explanation - GLONASS is an acronym, which stands for Globalnaya Navigazionnaya Sputnikovaya Sistema, or Global NavigationSatellite System. GLONASS is Russia’s version of GPS(Global Positioning System).
GPS developed by the USA has a network of 31 satellites covering this planet and has been widely used in commercial devices likemobile phones, navigators, etc.
GLONASS is developed by Russia originally started by Soviet Union in 1976. This has a network of 26 satellites covering the earth.
7. Consider the following about Pradhan Mantri Matru Vandana Yojana: 1. It is a Central Sector Scheme. 2. It provides conditional cash transfer to pregnant and lactating mothers who are not employed in any formal sector job under Central or State Government. 3. Minimum Age limit for getting benefits under is 19 Which of the following is Correct Option:
    A. 1, 2 and 3 B. 2 and 3 Only. C. 1 and 2 Only. D. 2 Only

Answer - Option B
Explanation - Pradhan Mantri Matritva Vandana Yojana (PMMVY), previously Indira Gandhi Matritva Sahyog Yojana (IGMSY), is a maternity benefit program run by the government of India.
PMMVY is implemented by the Ministry of Women & Child Development in collaboration with State Governments.
It is Centrally Sponsored Scheme under which the costsharing ratio between the Centre and the States & UTs with The legislature is 60:40, for the North-Eastern States & three Himalayan States, it is 90:10 and 100% Central assistance for Union Territories without Legislature.
It is a conditional cash transfer scheme for pregnant and lactating women of 19 years of age or above for first two live births.
It provides partial wage compensation to women for wage loss during childbirth and childcare and to provide conditions for safe delivery and good nutrition and feeding practices.
In 2013,the scheme was brought under the National Food Security Act, 2013 to implement the provision of cash maternity benefit of ?6,000 stated in the Act.
8. Consider the following statements about Small Finance Banks: 1. They must have a minimum paid up capital of Rs100 Crore. 2. They have a priority sector lending requirement of 75% of total adjusted net bank credit. 3. Minimum 50% of loans should be up to Rs25 Lakh. Which of the above statements is/are correct:
    A. 1 and 2 Only B. 2 and 3 Only C. 1 and 3 Only D. 1, 2 and 3

Answer - Option D
Explanation - The Reserve Bank of India has decided to allow urban cooperative banks (UCB) to convert into small finance banks (SFB), a move aimed at bringing these entities into mainstream banking. It has been decided to allow voluntary transition of UCBs meeting the prescribed criteria into SFBs.
What are the small finance banks? The small finance bank will primarily undertake basic banking activities of acceptance of deposits and lending to unserved and underserved sections including small business units, small and marginal farmers, micro and small industries and unorganized sector entities.
What they can do? Take small deposits and disburse loans.
Distribute mutual funds, insurance products, and other simple third-party financial products.
Lend 75% of their total adjusted net bank credit to the priority sector.
Maximum loan size would be 10% of capital funds to a single borrower, 15% to a group.
Minimum 50% of loans should be up to 25 lakhs.
What they cannot do?
Lend to big corporates and groups.
Cannot open branches with prior RBI approval for first five years.
Other financial activities of the promoter must not mingle with the bank.
It cannot set up subsidiaries to undertake non-banking financial services activities.
Can not be a business correspondent of any bank.
The guidelines they need to follow:
promoter must contribute a minimum 40% equity capital and should be brought down to 30% in 10 years.
Minimum paid-up capital would be Rs 100 Cr.
Capital adequacy ratio should be 15% of risk-weighted assets, TierI should be 7.5%.
Foreign shareholding capped at 74% of paid capital, FPIs cannot hold more than 24%.
Priority sector lending requirement of 75% of total adjusted net bank credit.
50% of loans must be up to Rs 25 lakh.
9. Consider the following statements about United Nation Human Right Council: 1. It came into existence with United Nations genesis itself 2. Members meet annually to discuss and overview the Human Rights situation in world. Which of the above statements is/are correct:
    A. 1 Only B. 2 Only C. 1 and 2 D. None

Answer - Option D
Explanation - The UN body was established in 2006 with the aim of promoting and protecting human rights around the globe, as well as investigating alleged human rights violations.
It is made up of 47 member states, which are selected by the UN General Assembly on a staggered basis each year for three-year-long terms.Members meet around three times a year to debate human rights issues and pass non-binding resolutions and recommendations by majority vote.
The council also carries out the Universal Periodic Review of all UN member states, which allows civil society groups to bring accusations of human rights violations in member states to the attention of the UN.
10. Consider the following statements about Social Media Communication Hub Project: 1. It will be handled by Broadcast Engineering Consultants India Ltd. which is a private sector entity. 2. The project is aimed at developing a new online platform for social interactions. Which of the above statements is/are correct:
    A. 1 Only B. 2 Only C. Both 1 and 2 D. None

Answer - Option A
Explanation - Public sector firm Broadcast Engineering Consultants India Ltd., which is supposed to be handling the Social media communication hub project, has extended the last date for tenders.
What is Social media communication hub?
The hub proposes to monitor social media (Facebook, Twitter, Instagram, and even email) handles at the very local level in multiple languages to carry out “sentiment analysis”, track down the influence-making social media users and to categorize the conversations on social media into positive, negative and neutral sections.
It also aimed to track real-time the way social media receives news on the government’s schemes and announcements and also political events.
As per the proposal, the project is meant to strengthen the social media division and recruit social media managers to be deployed in 712 districts of the country.
Each district will have one social media manager who will be entrusted with the tasks of keeping a close eye on the regional and local media, collecting data of regional media and of local events, providing content for social media and supporting media units at the regional level for social media publicity.
1. Consider the following statements about International Court of Arbitration: 1. It is a branch International Chambers of Commerce and is engaged in settling international commercial disputes. 2. It is headquartered in Geneva, Switzerland. Which of the above statements is/are correct:
    A. 1 Only B. 2 Only C. Both 1 and 2 D. None

Answer - Option A
Explanation -
The NITI Aayog and ICC International Court of Arbitration had recently organised a Workshop on Best Practices in International Arbitration in New Delhi.
The International Court of Arbitration is a branch of the International Chamber of Commerce (ICC) and one of the world’s leading institutions for providing international arbitration services. The International Court of Arbitration is known for resolving international commercial and business disputes, administering more than half of all arbitration disputes worldwide. The ICC seat is located in Paris.
2. Consider the following statements about International Monetary Fund: 1. It is paid by a Charge – known as quota – paid by member nations. 2. The function of the fund is to prevent and tackle financial crises. Which of the above statements is/are correct:
    A. 1 Only B. 2 Only C. Both 1 and 2 D. None

Answer - Option C
Explanation -
The IMF, along with the World Bank, was conceived in 1944 at a conference in Bretton Woods, in the US state of New Hampshire. It aims to preserve economic stability and to tackle – or ideally prevent – financial crises. Over time, its focus has switched to the developing world.
The IMF is funded by a charge – known as a “quota” – paid by member nations – based on a country’s wealth.
The IMF also acts as a lender of last resort, disbursing its foreign exchange reserves for short periods to any member in difficulties.
3. Consider the following statements about Monetary Policy Committee: 1. It comprises of 6 Members out of which 3 are nominated by government and 3 are from RBI. 2. Members of the Committee are appointed for 5 Years and are not eligible for reappointment. Which of the above statements is/are correct:
    A. 1 Only B. 2 Only C. Both 1 and 2 D. None

Answer - Option A
Explanation -
The committee will have six members. Of the six members, the government will nominate three. No government official will be nominated to the MPC. The other three members would be from the RBI with the governor being the ex-officious chairperson. Deputy governor of RBI in charge of the monetary policy will be a member, as also an executive director of the central bank.
Decision: Decisions will be taken by majority vote with each member having a vote.
RBI governor’s role: The RBI Governor will chair the committee. The governor, however, will not enjoy a veto power to overrule the other panel members, but will have a casting vote in case of a tie.
Selection: The government nominees to the MPC will be selected by a Search-cum-Selection Committee under Cabinet Secretary with RBI Governor and Economic Affairs Secretary and three experts in the field of economics or banking or finance or monetary policy as its members.
Term: Members of the MPC will be appointed for a period of four years and shall not be eligible for reappointment.
4. Consider the following statements about Udhyam Abhilasha: 1. It is a campaign to inspire youth to select entrepreneurship as preferred occupational choice. 2. The initiative is taken by SIDBI in collaboration with CSC e-Governance Services India Limited. Which of the above statements is/are correct:
    A. 1 Only B. 2 Only C. Both 1 and 2 D. None

Answer - Option C
5. Consider the following statements with reference to the Stockholm Convention: 1. Parties must take measures to restrict the production and use of the chemicals listed under Annex A. 2. Parties must take measures to reduce the unintentional releases of chemicals listed under Annex B into the environment. 3. Mercury is a persistent organic pollutant listed under this convention. Which of the statements given above is/are correct?
    A. 1 and 2 only B. 3 only C. 1, 2 and 3 D. None of the above

Answer - Option D
Explanation -
There’s an exclusive convention to address the adverse effects of mercury – the Minamata Convention.
Mercury is not listed under the Stockholm Convention.
POPs in Stockholm Convention:
Annex A (Elimination): Parties must take measures to eliminate the production and use of the chemicals listed under Annex A.
Specific exemptions for use or production are listed in the Annex and apply only to Parties that register for them. PCBs are listed in Annex A.
Annex B (Restriction): Parties must take measures to restrict the production and use of the chemicals listed under Annex B in light of any applicable acceptable purposes and/or specific exemptions listed in the Annex.
Annex C (Unintentional production): Parties must take measures to reduce the unintentional releases of chemicals listed under Annex C with the goal of continuing minimization and, where feasible, ultimate elimination.
6. Ratan Watal Committee sometimes seen in news is related to:
    A. Banking Reforms B. Reforms in Capital Market Governance in India C. Boosting MSME Sector D. Digital Payments.

Answer - Option D
7. Financial Stability and Development Council is Chaired by:
    A. Union Finance Minister B. Secretary, Department of Economic Affairs C. Secretary, Department of Financial Services D. Chief Economic Advisor, Ministry of Finance.

Answer - Option A
Explanation -
The Financial Stability and Development Council (FSDC) was constituted in December 2010.
The FSDC was set up to strengthen and institutionalize the mechanism for maintaining financial stability, enhancing inter-regulatory coordination and promoting financial sector development.
The Council is chaired by the Union Finance Minister and its members are Governor, Reserve Bank of India; Finance Secretary and/or Secretary, Department of Economic Affairs; Secretary, Department of Financial Services; Chief Economic Adviser, Ministry of Finance; Chairman, Securities and Exchange Board of India; Chairman, Insurance Regulatory and Development Authority and Chairman, Pension Fund Regulatory and Development Authority. It also includes the chairman of the Insolvency and Bankruptcy Board (IBBI). In May, the government through a gazette notification had included the ministry of electronics and information technology (MeitY) secretary in the FSDC in view of the increased focus of the government on the digital economy.
8. Consider the following statements with reference to selection of non-permanent members of the UNSC: 1. Once every five years, the General Assembly elects non-permanent members for a five-year term 2. There are ten non-permanent members 3. These seats are distributed on a regional basis Which of the statements given above is/are correct?
    A. 1 and 2 only B. 2 and 3 only C. 1 and 3 only D. 1, 2 and 3

Answer - Option B
Explanation -
Each year, the General Assembly elects five non-permanent members (out of ten in total) for a two year term. The ten non-permanent seats are distributed on a regional basis:
  • Five for African and Asian countries;
  • One for Eastern European countries;
  • Two for Latin American and Caribbean countries;
  • Two for Western European and other countries.

9. Consider the following statements about Pradhan Mantri Fasal Bima Yojana: 1. The scheme has been rolled out by subsuming multiple insurance schemes that were operational earlier. 2. Premium to be paid by farmers is 2% for both Rabi and Kharif Crops. 3. The scheme is voluntary in Nature. Which of the above statements is/are correct:
    A. 1 Only B. 1 and 2 Only C. 1 and 3 Only D. All are Correct

Answer - Option A
Explanation -
In April, 2016, the government of India had launched Pradhan Mantri Fasal Bima Yojana (PMFBY) after rolling back the earlier insurance schemes viz. National Agriculture Insurance Scheme (NAIS), Weather-based Crop Insurance scheme and Modified National Agricultural Insurance Scheme (MNAIS).
Premium: It envisages a uniform premium of only 2% to be paid by farmers for Kharif crops, and 1.5% for Rabi crops. The premium for annual commercial and horticultural crops will be 5%.
The scheme is mandatory for farmers who have taken institutional loans from banks. It’s optional for farmers who have not taken institutional credit.
10. Consider the following statements about GI Tag: 1. It is given only to agricultural and Natural product. 2. GI tag is covered as an element of Intellectual Property Right under Paris Convention for Protection of Industrial Policy. Which of the above statements is/are correct:
    A. 1 Only B. 2 Only C. Both 1 and 2 D. None

Answer - Option B
Explanation -
What is it?
A GI is primarily an agricultural, natural or a manufactured product (handicrafts and industrial goods) originating from a definite geographical territory.
Significance of a GI tag:
Typically, such a name conveys an assurance of quality and distinctiveness, which is essentially attributable to the place of its origin.
Security:
Once the GI protection is granted, no other producer can misuse the name to market similar products. It also provides comfort to customers about the authenticity of that product.
Provisions in this regard:
GI is covered as an element of intellectual property rights (IPRs) under Paris Convention for Protection of Industrial Property.
At the international level, GI is governed by WTO’s Agreement on Trade-Related Aspects of Intellectual Property Rights (TRIPS).
In India, Geographical Indications of Goods (Registration and Protection Act), 1999 governs it.
1. UN Investment Promotion Award are given annually to honour the investment promotion agencies world over. This year’s award was won by Indian Body. It is:
    A. Department of Industrial Policy and Promotion. B. Federation of Indian Chambers of Commerce and Industry. C. Ministry of Corporate Affairs. D. Invest India

Answer - Option D
Explanation -
Invest India, the country’s investment promotion body, has won United Nations (UN) Award for excellence in promoting investments in sustainable development.
2. Consider the following statements about Tea Board of India: 1. It is a statutory body functioning under Tea Act 1953. 2. It comes under the Jurisdiction of Ministry of Commerce. Which of the above statements is/are correct:
    A. 1 Only B. 2 Only C. Both 1 and 2 D. None

Answer - Option C
Explanation -
About Tea Board of India:
The Tea Board is set up under the Tea Act 1953.
The Tea Board is functioning as a statutory body of the Central
Government under the Ministry of Commerce.
The Board is constituted of 31 members (including Chairman) drawn from Members of Parliament, tea producers, tea traders, tea brokers, consumers, and representatives of Governments from the principal tea producing states, and trade unions. The Board is reconstituted every three years.
Functions:
The Tea Board India is responsible for the assignment of certification numbers to exports of certain tea merchants. This certification is intended to ensure the teas’ origin, which in turn would reduce the amount of fraudulent labeling on rare teas.
The Tea Board India’s tasks include endorsement of the diverse production and productivity of tea, financial support of research organizations and the monitoring of advances in tea packaging as it relates to health beneficial aspects.
It coordinates research institutes, the tea trade, and government bodies, ensuring the technical support of the tea trade in the global industry.
3. Which of the following states had become World’s first totally organic agriculture state?
    A. Himachal Pradesh B. Uttrakhand C. Kerala D. Sikkim

Answer - Option D
Explanation -
Sikkim has been awarded UN Food and Agriculture Organisation’s (FAO) Future Policy Gold Award (Gold Prize) for its achievement in becoming the world’s first totally organic agriculture state.
Future Policy Gold Award:
The prizes, nicknamed the “Oscar for best policies”, honour exceptional policies adopted by political leaders who have decided to act, no longer accepting widespread hunger, poverty or environmental degradation.
Previously it was honoured for policies combating desertification, violence against women and girls, nuclear weapons and pollution of the oceans.
4. With reference to ‘Ekalavya Model Residential School (EMRS)’, which of the following statement is/are correct? 1. It is a Government of India scheme for a model residential school for Indian tribals 2. It provides for free residential facilities to all poor students. 3. It supports the construction of infrastructure that fulfills education and cultural needs of student life. Select the correct answer using the code given below.
    A. 2 only B. 1 and 2 only C. 2 and 3 only D. 1 and 3 only

Answer - Option D
Explanation -
Eklavya model residential school (EMRS)
Statement 1 is correct: It is a Government of India scheme for model residential school for Indian tribals (Scheduled Tribes, ST) across India. It was established by the Tribal Ministry. The residential schools aim to provide quality middle and high-level education to tribal children in remote areas of the country. It overlooks the physical, mental and socially relevant development of all students enrolled in each and every EMRS. Students will be empowered to be change agent, beginning in their school, in their homes, in their village and finally in a larger context. Focus differentially on the educational support to be made available to those in Standards XI and XII, and those in standards VI to X, so that their distinctive needs can be met, Support the annual running expenses in a manner that offers reasonable remuneration to the staff and upkeep of the facilities.
Statement 2 is incorrect: It does not provide free residential facilities to all poor students. (Only to Tribal Students)
Statement 3 is correct: Support the construction of infrastructure that fulfills education, physical, environmental and cultural needs of student life. The government aims to build at least one Eklavya model residential school (EMRS) in every block with 50 percent tribal population and cover at least 20,000 tribal people by 2022 so that the tribal population can access education in areas where they are located, according to the 2018 budget announcement.
Smart solution:
Statement 2 says it provides for free residential facilities to all poor students which are incorrect; rather it is for all tribal students. Hence, option a, b and c can be easily eliminated.
In News:
The Ministry of Tribal Affairs has sought more funds from the Ministry of Finance for its Eklavya model residential schools.
5. Which of the following statements is/are not correct about DNA Based Technology Use and Regulation) Bill, 2017? 1. It proposes setting up of a DNA Data Bank as Statutory Authority. 2. Bill restricts DNA profiling to the specific purpose of identification of a person only. 3. India would the first country to have such a law. Select the correct answer using the code given below:
    A. 1 and 2 only B. 3 only C. 3 and 4 Only D. 1 and 4 Only

Answer - Option B
Explanation -
The draft Bill named ‘The DNA Based Technology (Use and Regulation) Bill, 2017’ is drafted after examining various judicial pronouncements and constitutional provisions. According to the Draft bill, DNA profiling is to be used for disaster victim identification, investigation of crimes, identification of missing persons and human remains and for medical research purposes. The Bill provides for the setting up of a statutory DNA Profiling Board to spell out procedures and standards to establish DNA laboratories.
Statement 2 is correct: Draft Bill restricts DNA profiling to the specific purpose of identification of a person and not for extracting other information.
Statement 1 is correct: It provides for the creation of DNA data banks, at national and regional levels, which would be responsible for storing DNA profiles received from the accredited laboratories. It also provides for sharing of DNA profiles with foreign governments or other government agencies, organizations or institutions would only be for the purpose specified in this Act or any of its agencies, including identification of missing persons, disaster victims, suspects. Any violation would lead to imprisonment, which may extend up to three years, and a fine which may extend up to ₹2 lakh. The Bill gives the right to an undertrial to request for another DNA test in case of doubts that his earlier samples may have been contaminated.
Statement 3 is incorrect: India is not the first country to have such a law.
6. Consider the following statements about Zika Virus: 1. It is transmitted by the same mosquito that transmits dengue, chikungunya and yellow fever. 2. It was first identified in humans and later in animals. Which of the above statements is/are correct?
    A. 1 only B. 2 only C. Both 1 and 2 D. Neither 1 nor 2

Answer - Option A
Explanation -
Zika Virus
Zika virus is a mosquito-borne disease caused by a virus transmitted primarily by Aedes mosquitoes, which bite during the day.
Statement 1 is correct: Zika virus is primarily transmitted by the bite of an infected Aedes mosquito, in tropical and subtropical regions. This is the same mosquito that transmits dengue, chikungunya and yellow fever.
Statement 2 is incorrect: It was first identified in Uganda in 1947 in monkeys. It was later identified in humans in 1952 in Uganda and the United The Republic of Tanzania. Zika virus is also transmitted from mother to fetus during pregnancy, through sexual contact, transfusion of blood and blood products, and organ transplantation. There is no treatment nor vaccine available for Zika virus infection or its associated diseases.
In News:
Indian Council of Medical Research surveillance system has recently detected cases of Zika virus in Jaipur.
7. The terms LUBAN, TITLE, and SAGAR, often seen in the news, are related to:
    A. Submarines B. Cyclones C. Missiles D. Research Stations at Antarctica

Answer - Option B
8. With reference to ‘Comprehensive Convention on International Terrorism’ (CCIT)‘, consider the following statements: 1. India is against the adoption of CCIT. 2. Negotiations for this treaty held at United Nations General Assembly. Which of the statements given above is/are INCORRECT?
    A. 1 only B. 2 only C. Both 1 and 2 D. Neither 1 nor 2

Answer - Option A
Explanation -
The Comprehensive Convention on International Terrorism is a proposed treaty which intends to criminalize all forms of internal terrorism and deny terrorists, their financiers and supporters access to funds, arms, and safe havens. Statement 1 is incorrect: India has been advocating early adoption of CCIT and has called the global community to promote the anti-terror legal framework and execute the global counter-terrorism strategy in an integrated manner. India is of the view that comprehensiveness of global counter-terrorism strategy will not be concluded devoid of the conclusion of the CCIT which is at an important phase of discussion before the UN.
Statement 2 is correct: The negotiations for this treaty are currently under way at the United Nations General Assembly’s Ad Hoc Committee established by Resolution 51/210 of 17 December 1996 on Terrorism and the United Nations General Assembly Sixth Committee (Legal). Although consensus eludes towards adoption of the terrorism convention, discussions have yielded three separate protocols that aim to tackle terrorism: International Convention for the Suppression of Terrorist Bombings adopted on 15 December 1997; International Convention for the Suppression of the Financing of Terrorism, adopted on9 December 1999; and International Convention for the Suppression of Acts of Nuclear Terrorism adopted on 13 April 2005.
9. IND-INDO CORPAT is a:
    A. Joint exercise between India and Indonesia B. Corporate Summit between India and ASEAN> C. Partnership between India & Indonesia for make in India D. Security Dialogue between Indian Ocean Countries

Answer - Option A
Explanation - The Indian Navy and Indonesian Navy have been carrying out coordinated patrols (CORPAT) twice a year since 2002. The CORPAT is also accompanied by a bilateral exercise at the sea.
10. Which of the following Country has initiated the Artificial Moon Project?
    A. China B. United States of America C. India D. Norway

Answer - Option A
Explanation -
Artificial Moon Project
Option (a) is correct: Chinese scientists plan to send three artificial moons into space in the next four years, and the moons —
made from reflective material like a mirror — are expected to orbit at 500 kilometres above the Earth and light up an area with a diameter of 10 to 80 kilometres.
The artificial moon will have a reflective coating that can reflect sunlight back to Earth, similar to how the moon shines.
The illuminated satellite is said to be eight times brighter than the real moon. The satellites’ brightness and service time are both adjustable, and the accuracy of the lighting can be controlled within tens of meters.
The three artificial moons would operate alternately in order to significantly reduce infrastructural electricity consumption, especially during winter. The illuminated satellite is designed to complement the moon at night.
In News:
China is in the process of creating an “artificial moon” that would be bright enough to replace the streetlights in the south-western city of Chengdu by 2020.
1. Consider the following statements 1. S 400 Triumf is a long-range air defense missile system. 2. Kamov 226T is a light multipurpose helicopter. 3. The Kamov 226T will replace the Cheetah and Chetak choppers. Which of the above statements is/are correct?
    A. 1 only B. 2 and 3 only C. 1 and 2 only D. 1, 2 and 3

Answer - Option D
Explanation -
S 400 Triumf Air Defense System
Statement 1 is correct: It is one of the most advanced long-range air defence missile systems in the world. It is referred to as SA-21 Growler by NATO. It is an upgraded version of the S-300 defense system and is manufactured by Almaz-Antey. It has been in service in Russia since 2007. It is capable of firing three types of missiles. Thus, it creates a layered defense and simultaneously engaging 36 targets. It has the capability to destroy incoming all airborne targets at ranges of up to 400 km. The five S-400 systems will be operated by the Indian Air Force (IAF). India is the second export customer, after China to purchase this most advanced defense system.
Ka-226T helicopters:
Statement 2 and 3 are correct: Kamov 226T is a light multipurpose helicopter designed for work in difficult conditions like high mountains, hot climate and marine areas. It allows for reconnaissance, monitoring, targeting, and transportation. It also boasts of incredibly precise hovering ability, excellent maneuverability, and high safety standards. The helicopters will be made by Hindustan Aeronautics Limited (HAL) with Russian cooperation under Make in India. It will replace the aging Cheetah and Chetak choppers. After induction, Kamov helicopters will operate at 20,000 feet and also help to maintain the forward posts on the Siachen glacier.
In News: India has signed a defense deal worth nearly Rs 39,000 crore with Russia to purchase five states of art S-400 ‘Triumph’ air defense systems. Besides, India also announced to collaborate in making four stealth frigates and setting up a joint production facility for making Kamov helicopters under Make in India.
2. Which of the following statements is/ are correct about NASA-ISRO Synthetic Aperture radar satellite (NISAR)? 1. It is the first satellite mission to use two different Radar frequencies. 2. It is the world’s most expensive earth imaging satellite. 3. It is designed to observe Earthquake tsunamis, volcanoes and landslides. Select the correct answer using the code given below:
    A. 1 only B. 1 and 3 only C. 1 and 2 Only D. All are Correct

Answer - Option D
Explanation -
NASA-ISRO Synthetic Aperture radar satellite (NISAR):
Statement 1 is correct: NISAR is a dual frequency (L & S Band) Radar Imaging Satellite. It will be the first radar imaging satellite to use dual frequency. In this joint mission, NASA’s Jet Propulsion Laboratory (JLP) will be responsible for the design and development of L-band SAR, GPS system and data recorder. ISRO will be responsible for the design and development of S-band SAR, Spacecraft Bus, data transmission system, spacecraft integration & testing, launch using GSLV and on-orbit operations. It is expected to be launched in the year 2021. It will have a mission life of 3 years. The satellite is planned to be launched into Sun-synchronous dawn to dusk orbit. It will provide an unprecedented detailed view of Earth by taking snapshots every week using advanced radar imaging.
Statement 3 is correct: It is designed to observe and take measurements of some of the planet’s most complex processes, including ecosystem disturbances, natural hazards such as earthquakes, tsunamis, volcanoes and landslides.
3. The Nobel Peace Prize for 2018 has been awarded:
    A. To end the use of sexual violence as a weapon of war. B. To end the menace of Child Trafficking. C. For efforts to stop internal Civil Uprising. D. For building consensus among different stakeholders to minimize the impact of Global Warming.

Answer - Option A
Explanation -
The Nobel Peace Prize for 2018 has been awarded to Congolese gynecologist Dr. Denis Mukwege and Yazidi human rights activist Nadia Murad for their efforts to end the use of sexual violence as a weapon of war and armed conflict. Denis Mukwege is the helper who has devoted his life to defending victims of wartime sexual violence. Fellow laureate Nadia Murad is the witness who tells of the abuses perpetrated against herself and others. Dr. Denis Mukwege has spent large parts of his adult life helping the victims of sexual violence in the Democratic Republic of Congo. Dr.Mukwege and his staff have treated thousands of patients who have fallen victim to such assaults. Ms. Murad is one of an estimated 3,000 Yazidi girls and women who were victims of rape and other abuses by the Islamic State. She has shown uncommon courage in recounting her own sufferings and speaking up on behalf of other victims. The panel had received nominations for 216 individuals and 115 organizations. But only a few dozen of them are known, since the committee keeps the list of nominations secret for 50 years, although some candidates are revealed by their nominators. The 2018 prize is worth 9 million Swedish kronor ($1.01 million). Past winners who came under criticism include former U.S. President Barack Obama, who won in 2009 after less than a year in office, and Myanmar leader Aung San Suu Kyi
4. What is Train 18, recently seen in the news?
    A. First intercity Engineless Train in India with 18 coaches. B. 18th anniversary of launching Double Decker Train in India. C. A Train for Youth (above 18 years) recently approved by Government of India. D. A Heritage Train launched by Indian Railways in 2018.

Answer - Option
Explanation -
Train 18:
Train 18 is an Indian semi-high speed train and India’s first intercity electric multiple units, a type of self-propelled engine-less train. It is also the first train to be designed and built entirely in India. It was designed and built by Integral Coach Factory (ICF) Chennai under the Indian government’s Make in India Initiative over a span of 18 months. It is energy efficient as its coaches will be fitted with LED lights and are provided with bio-toilets. The first set of trains will run between New Delhi and Bhopal, replacing the Delhi–Bhopal Shatabdi Express, which has been running for thirty years and reducing travel time along the route by 15 percent, or even more once the tracks are properly fitted for the unit.
5. Consider the following statements about Parker Mission: 1. It is a Mission to explore Sun’s Corona. 2. It is the first NASA mission that has been named after a living individual. Which of the above statements is/are correct?
    A. 1 only B. 2 only C. Both 1 and 2 D. Neither 1 nor 2

Answer - Option C
Explanation -
Parker Solar Probe:
Statement 1 is correct: Parker Solar Probe will swoop to within 4 million miles of the sun’s surface, facing heat and radiation like no Launching in 2018, Parker Solar Probe will provide new data on solar activity and make critical contributions to our ability to forecast major space-weather events that impact life on Earth. Parker Solar Probe is a mission to unlock the mysteries of the corona, but also to protect a society that is increasingly dependent on technology from the threats of space weather. spacecraft before.
6. Chandra X-ray Observatory is a:
    A. Telescope by NASA B. X-ray detection centre by ISRO C. Observatory by European Space Agency D. Space observatory by India and Japan

Answer - Option A
Explanation -
The Chandra X-Ray Observatory is a NASA telescope that looks at black holes, quasars, supernovas, and the like – all sources of high energy in the universe. It shows a side of the cosmos that is invisible to the human eye. It was previously known as the Advanced X-ray Astrophysics Facility (AXAF). After more than a decade in service, the observatory has helped scientists glimpse the universe in action. It has watched galaxies collide, observed a black hole with cosmic hurricane winds, and glimpsed a supernova turning itself inside out after an explosion. The telescope is named after the Nobel Prize-winning Indian-American astrophysicist Subrahmanyan Chandrasekhar.
7. The ‘Global Guidelines on Sanitation and Health’ have been launched by which of the following?
    A. The United Nations International Children’s Emergency Fund (UNICEF) B. United Nations Development Programme (UNDP) C. World Health Organization (WHO) D. World Bank (WB)

Answer - Option C
Explanation -
The Global Guidelines On Sanitation and Health: World Health Organization (WHO) launched its first Global Guidelines on Sanitation and Health.
The new guidelines set out four principal recommendations: Sanitation interventions should ensure that the entire community has access to toilets that safely contain excreta. The full sanitation system should undergo local health risk assessments to protect individuals and communities from exposure to excreta – whether this is from unsafe toilets, leaking storage or inadequate treatment. Sanitation should be integrated into regular local government-led planning and service provision to avert the higher costs associated with retrofitting sanitation and to ensure sustainability. The health sector should invest more and play a coordinating role in sanitation planning to protect public health.
8. Consider the following statements: 1. Inland Waterways Authority of India (IWAI) will be starting a new Roll on-Roll off (Ro-Ro) facility for Majuli Island. 2. Majuli is one of the biggest riverine islands in the world located on the Teesta river. Which of the statements given above is/are incorrect?
    A. 1 only B. 2 only C. Both 1 and 2 D. Neither 1 nor 2

Answer - Option C
Explanation -
Inland Waterways Authority of India (IWAI)
Statement 1 is correct: Inland Waterways Authority of India (IWAI) will be starting a new Roll on-Roll off (Ro-Ro) facility in collaboration with the Government of Assam to provide the much-needed connectivity for Majuli Island.
Statement 2 is incorrect: Majuli is one of the biggest riverine islands in the world located on river Brahmaputra and faces serious challenges of connectivity. It has 144 villages with a population of over 1,50,000. The commencement of Ro-Ro services to Majuli Island would be a landmark event towards augmenting connectivity not only in Assam but the entire North Eastern Region. Currently, there are only four road bridges across river Brahmaputra - at Jogighopa, Guwahati, Tezpur, and Sadiya for connectivity between southern and northern parts of Assam. People residing on either side of the river need to cross the river using conventional ferry service at various locations for their day to day needs. In the absence of an adequate number of bridges, cargo and passenger movement takes through longer road routes leading to critical loss of time and money.
9. With reference to ‘World agriculture Prize’, which of the following statements is/are correct? 1. The prize has been awarded to Professor M.S. Swaminathan for the year 2018. 2. It is an annual prize presented to only individuals from Asian, African or Latin American countries. 3. This is the second time any Indian has been awarded two times in a row. Select the correct answer using the code given below:
    A. 1 only B. 1 and 3 only C. 2 and 3 only D. 1, 2 and 3

Answer - Option A
Explanation -
Statement 1 is correct: Professor M.S. Swaminathan, founder of the M.S. Swaminathan Research Foundation (MSSRF) was awarded the first World Agriculture Prize at the 11th Global Agriculture Leadership Summit in New Delhi at the event hosted by the Indian Council of Food and Agriculture (ICFA). Recognized worldwide for his basic and applied research in genetics, cytogenetics, radiation, and chemical mutagenesis, food and biodiversity Environment Programme as ‘The Father of Economic Ecology’ owing to his commitment towards the ever-green revolution movement in agriculture.
The M.S. Swaminathan Junior Research Fellowship — to be awarded from 2019 onwards — was also instituted on the occasion. The fellowship would help promote linkages between agriculture, nutrition, and health, through a ‘Farming Systems for Nutrition’ program. Further, the cash associated with the prize would be used for creating fellowships at MSSRF at the postgraduate level, to convert the United Nations Sustainable Development Goals into field-level accomplishments. The fellowship would be an opportunity for young scholars to address the UN Sustainable Development Goals, including the eradication of hunger and climate change. The five areas considered for the
fellowship are: the UN Sustainable Development Goal 2 of Zero Hunger, Farming System for Nutrition (FSN), genetic garden of biofortified crops and halophytes, bio-valley for curative and culinary diversity and eco-technologies for eco-enterprises.
Statement 3 is incorrect: The prize has been instituted by the ICFA with an aim to recognize individuals who have served humanity through agriculture.
10. With reference to Vayoshreshtha Samman, consider the following statements: 1. The Ministry of Social Justice and Empowerment confers this award. 2. The award is given to eminent or outstanding Institutions or organisations and individuals. Which of the above statements is/are incorrect?
    A. 1 only B. 2 only C. Both 1 and 2 D. Neither 1 nor 2

Answer - Option C
Explanation -
Statement 1 is correct: The Ministry of Social Justice and Empowerment in the year 2005 to confer Vayoshreshtha Sammans to eminent senior citizens and Institutions in recognition for their service to elderly persons, especially indigent senior citizens. The “Vayoshrestha Samman” was elevated to the level of national awards in 2013 under 13 categories.
Statement 2 is correct: The award would be given to eminent or outstanding Institutions or organisations and individuals from any part of the country.
1. International Day for Disaster Reduction is celebrated on:
    A. 25 October B. 13 October C. 6 October D. 30 October

Answer - Option B
Explanation -
The theme of the 2018 International Day for Disaster Reduction was ‘Reducing Disaster Economic Losses’. The 2018 theme continues as part of the “Sendai Seven” campaign, centered on the seven targets of the Sendai Framework. This year focuses on Target C of the Sendai Framework, which is, ‘reducing disaster economic losses in relation to global GDP by 2030’.
2. Living Planet Report 2018 was released by:
    A. Green Climate Fund B. United Nation Environment Programme C. World Wide Fund for Nature D. None of the above

Answer - Option C
3. Consider the following statements: 1. New Horizon is the European Space Agency’s spacecraft to map the surfaces of Pluto including its atmosphere. 2. It recently sent back data suggesting that there could be an Oxygen Wall at the end of our solar system. Which of the above statements is/are correct:
    A. 1 Only B. 2 Only C. Both 1 and 2 D. None

Answer - Option D
Explanation -
Scientific data sent back by National Aeronautics and Space Administration’s (NASA) New Horizons spacecraft suggests that there could be a hydrogen wall at the end of our solar system. New Horizons was launched on 19 January 2006, and has been travelling through space for the past nine years. The mission will complete what NASA calls the reconnaissance of the classical solar system, and it makes the U.S. the first nation to send a space probe to every planet from Mercury to Pluto. The probe has traveled more than 3 billion miles to reach Pluto New Horizon’s core science mission is to map the surfaces of Pluto and Charon, to study Pluto’s atmosphere and to take temperature readings.
4. Consider the following statements about Horizon 2020: 1. It is a European Union Research and Innovation program. 2. The program is open Only to the Member States of the European Union. Which of the above statements is/are correct:
    A. 1 Only B. 2 Only C. Both 1 and 2 D. None

Answer - Option A
Explanation -
Horizon 2020 is the biggest EU Research and Innovation programme ever with nearly €80 billion of funding available over 7 years (2014 to 2020) – in addition to the private investment that this the money will attract. It promises more breakthroughs, discoveries, and world- firsts by taking great ideas from the lab to the market.
Horizon 2020 is open to everyone, with a simple structure that reduces red tape and time so participants can focus on what is really important The European Union and India will collaborate in research and innovation for developing a next generation influenza vaccine to protect people worldwide. The EU is funding is under its program for research and innovation ‘Horizon 2020’. The EU and the Indian government’s Department of Biotechnology have committed 15 million Euros each to fund this joint project.
5. Consider the following statements with reference to the Centres for Fourth Industrial Revolution: 1. They have been established under the aegis of the World Economic Forum. 2. There are four such centers established across the world. 3. In India, Bangalore hosts the fourth such center. Which of the statements given above is/are correct ?
    A. 1 and 2 only B. 3 only C. 1 and 3 only D. 1, 2 and 3

Answer - Option A
Explanation -
The WEF established the first Centre for the Fourth Industrial Revolution Network in March 2017 to help co- esign creative solutions to address the challenges associated with emerging technologies. The Centre for the Fourth Industrial Revolution Network currently has offices in San Francisco (HQ), Tokyo, Beijing and most recently, Mumbai, and is continuing to expand around the globe. The Mumbai centre has selected drones, artificial intelligence and blockchain as the first three project areas.
6. IBSAMAR is a joint maritime exercise involving: 1. India 2. Brazil 3. South Africa 4. Myanmar 5. Russia Which of the above statements is/are correct:
    A. 1, 2 and 3 B. 2, 3 and 5 C. 1 and 3 D. All of the above

Answer - Option A
Explanation -
The sixth edition of IBSAMAR, a joint multi–national maritime exercise between Indian, Brazilian and South African Navies was held at Simons Town, South Africa.
Aim: The aim of this maritime exercise is to undertake collective training for participating navies, building interoperability and mutual understanding as well as sharing of best practices.
7. Exercise AVIAINDRA-18 was a Biennial Air Force exercise between India and:
    A. Russia B. Rawanda C. Japan D. Japan

Answer - Option A
8. Recently, India for Humanity Initiative was launched by:
    A. Ministry of Social Justice and Empowerment B. Ministry of Health and Family Welfare C. Ministry of External Affairs D. Ministry of Culture

Answer - Option C
Explanation -
Union Ministry of External Affairs (MEA) has launched India for Humanity initiative to commemorate the 150th birth anniversary of Mahatma Gandhi and honour of his service to humanity. It will feature year-long series of artificial limb fitment camps in a number of countries spanning globe. For this initiative, MEA has collaborated with renowned charitable organisation Bhagwan Mahaveer Viklang Sahayata Samiti (BMVSS).
Aim: To provide physical, economic and social rehabilitation of differently- become self- respecting and productive members of society. It focuses on Mahatma Gandhi’s philosophy of compassion, caring and service to humanity abled around the world by helping them regain their mobility and dignity to
9. #MeToo movement was inititated by:
    A. Tarana Burke B. Harvey Weinstein C. Alyssa Milano D. Gwyneth Paltrow

Answer - Option A

Other Articles



shape Job-Alerts

Competitive Exams - Recent Job Notifications
Category
Banking SSC Railway
Defence Police Insurance
Click Here For – All India Latest Jobs

shape SP Quiz

Competitive Exams - Practice Sets
Category Quiz
English Language Spotting Errors
Current Affairs Current Affairs
Quantitative Aptitude Data Interpretation

shape GK

General Knowledge for Competitive Examinations
Topic Name of the Article

GK - World
Women Leaders
Tallest Residential Buildings

GK - India
FSSAI
Social and Religious Movements

GK - Abbreviations
Indian Railways Acronyms
International Organizations Abbreviations

GK - Banking & Insurance
Banking Abbreviations & Finance Abbreviations
Finance Abbreviations


GK - Science & Technology
Energy Transformation Devices
Web Portals
Indian Satellites